You have been given 3.152 g of copper from which you are to fashion a wire whose resistance is 2,473.562 ohms. The density of copper is 8900 kg/m 3 and the resistivity of copper is 1.7 x 10 -8 ohm m. What will the diameter, in mm, of the wire be that you fashion?

Answers

Answer 1

Given:

Mass of copper, m = 3.152 g = 0.003152 kg

Resistance = 2,473.562 ohms

Density of copper = 8900 kg/m³

Resistivity of copper = 1.7 x 10⁻⁸ ohm.m

Let's find the diameter of the wire.

Apply the density formula to find the volume:

[tex]\begin{gathered} \rho=\frac{m}{v} \\ \\ v=\frac{m}{\rho}=\frac{0.003152}{8900} \\ \\ v=3.54*10^{-7}\text{ m}^3 \\ \\ (\pi r^2)l=3.54*10^{-7}\text{ m}^3 \end{gathered}[/tex]

Now, apply the formula:

[tex]\begin{gathered} R=\frac{\rho l}{A} \\ \\ l=\frac{RA}{\rho} \\ \\ l=\frac{2473.562*\pi r^2}{1.7*10^{-8}} \\ \\ \frac{l}{\pi r^2}=\frac{2473.562}{1.7*10^{-8}} \\ \\ \frac{l}{\pi r^2}=1.455*10^{11} \\ \\ l=1.455*10^{11}*\pi r^2 \end{gathered}[/tex]

Now, combine both expressions for L:

[tex]1.455*10^{11}*\pi r^2*\pi r^2=3.54*10^{-7}[/tex]

Solving further:

[tex]\begin{gathered} (\pi r^2)^2=\frac{3.54*10^{-7}}{1.455*10^{11}} \\ \\ (\pi r^2)=\sqrt{2.434*10^{-18}} \\ \\ \pi r^2=1.56*10^{-9} \\ \\ r^2=\frac{1.56*10^{-9}}{\pi} \\ \\ r=\sqrt{\frac{1.56*10^{-9}}{\pi}} \\ \\ r=2.228*10^{-5} \end{gathered}[/tex]

r is the radius.

We know that:

Diameter = 2 x radius

Thus, we have:

[tex]\begin{gathered} d=(2.228*10^{-5})*2 \\ \\ d=4.4569*10^{-5}\text{ m} \\ \\ d=0.044569*10^{-3}m \\ \\ d=0.0446\text{ mm} \end{gathered}[/tex]

Therefore, the diameter in mm will be 0.0446 mm

ANSWER:

0.0446 mm


Related Questions

A deuteron particle consists of one proton and one neutron and has a mass of 3.34x10^-27 kg. A deuteron particle moving horizontally enters uniform, vertical 0.0800 T magnetic field and follows a circular arc of radius of 38.5 cm. What would be the radius of the arc followed by a proton that entered the field with the same velocity as the deuterium?

Answers

_dThe radius of curvature of a subatomic particle under a magnetic field is given by the following formula:

[tex]r=\frac{mv}{qB}[/tex]

Where:

[tex]\begin{gathered} r=\text{ radius} \\ v=\text{ velocity} \\ q=\text{ charge} \\ B=\text{ magnetic field} \end{gathered}[/tex]

We can determine the quotient between the velocity and the charge of the deuteron particle from the formula. First, we divide both sides by the mass:

[tex]\frac{r_d}{m_d}=\frac{v}{q_B_}[/tex]

Now, we multiply both sides by the magnetic field "B":

[tex]\frac{Br_d}{m_d}=\frac{v}{q}[/tex]

Since the charge of the deuterion is the same as the charge of the proton and the velocity we are considering are the same this means that the quotient between velocity and charge is the same for both particles. Therefore, we can apply the formula for the radius again, this time for the proton:

[tex]r_p=\frac{m_pv}{qB}[/tex]

And substitute the quotient between velocity and charge:

[tex]r_p=\frac{m_p}{B}(\frac{Br_d}{m_d})[/tex]

Now, we cancel out the magnetic field:

[tex]r_p=\frac{m_pr_d}{m_d}[/tex]

Now, we substitute the values:

[tex]r_p=\frac{(1.67\times10^{-27}kg)(0.385m)}{(3.34\times10^{-27}kg)}[/tex]

Solving the operations:

[tex]r_p=0.193m=19.3cm[/tex]

Therefore, the radius is 19.3 cm.

For an object starting from rest and accelerating with constantacceleration, distance traveled is proportional to the square of thetime. If an object travels 2.0 furlongs in the first 2.0 s, how far willit travel in the first 4.0 s?

Answers

Since the object is accelerating with constant acceleration we can use the following formula for the position of the object:

[tex]x=x_0+v_0t+\frac{1}{2}at^2[/tex]

where x0 is the initial position, v0 is the initial velocity, a is the acceleration and t is the time. In this case, the initial position and velocity are zero. Plugging the values given we have:

[tex]\begin{gathered} 2=\frac{1}{2}a(2)^2 \\ 2=2a \\ a=1 \end{gathered}[/tex]

Hence, the acceleration of the object is 1 furlong per second per second.

Once we know the acceleration we can use the same formula to determine how far the object will travel in four seconds.

[tex]\begin{gathered} x=\frac{1}{2}(1)(4)^2 \\ x=\frac{16}{2} \\ x=8 \end{gathered}[/tex]

Therefore, the object will travel 8 furlongs in four seconds.

If the man and woman are taken to a planet where the acceleration due to gravity is twice that of earth repeat the woman mass was 25kg on earth and the man was 300N on another planet

Answers

The mass of the woman is 25 kg because the mass is constant.

The mass of the man can be found using the formula: W = mg, where g is double Earth's gravity.

[tex]\begin{gathered} m=\frac{W}{g} \\ m=\frac{300N}{2\cdot9.8\cdot\frac{m}{s^2}} \\ m=\frac{300}{19.6}kg \\ m\approx15.3\operatorname{kg} \end{gathered}[/tex]

The mass of the man is 15.3 kg.

The weight of the man on Earth can be found with the same formula but using Earth's gravity.

[tex]\begin{gathered} W=15.3\operatorname{kg}\cdot9.8\cdot\frac{m}{s^2} \\ W=149.94N \end{gathered}[/tex]

The weight of the man on Earth is 149.94 N.

At last, the weight of the woman on Earth can be found using the same method before.

[tex]\begin{gathered} W=25\operatorname{kg}\cdot9.8\cdot\frac{m}{s^2} \\ W=245N \end{gathered}[/tex]

The weight of the woman on Earth is 245N.

I think this is all statements are true but I just want to make sure

Answers

Given that a bug flies into the windshield of a car going. Let's select the correct statements.

According the NEwton's third law, the force exerted on the bug by the car is equal to the force extered on the car by the bug.

To determine the acceleration, we have:

[tex]a=\frac{F}{m}[/tex]

Where:

F is the force

m is the mass

The mass of the car will be greater than the mass of the bug.

Since the mass of the car is greater than the mass of the bug and they have the same force, we can say the acceleration of the bug is greater than the acceleration of the car.

Statement B is correct.

The force of impact is the same for both according to Newton's third Law.

Both the car and the bug deliver the same magnitude of impulse on each other.

Therefore, all statements are correct.

ANSWER:

All statements are true.

URGENT!! ILL GIVE
BRAINLIEST!!!! AND 100 POINTS!!!!!!

A feather and a bowling ball are each dropped from an equal height in a vacuum and land at the same time. Which graph shows the total mechanical energy of the bowling ball as it falls?

Answers

The total mechanical energy of the bowling ball and the feather is shown by the graph in option D

What is the total mechanical energy?

We know that the mechanical energy is the energy that is possessed by a body by virtue of its motion or by virtue of its staying at a place. Thus mechanical energy is possessed by an object that is moving or by an object that is at rest.

In this case, we have a  feather and a bowling ball are each dropped from an equal height in a vacuum and land at the same time. We know that the mechanical energy of the two objects must be constant. This is because, the potential energy of the feathers and the ball at a height is converted to kinetic energy as the two objects begin to move.

The graph that would show the total energy must be one in which the energy axis of the graph is constant as we see in option D.

Learn  more about mechanical energy:https://brainly.com/question/13552918

#SPJ1

Answer:

the answer is D

Explanation:

explain how the intensity of the UV light vaires across the Earth

Answers

Some factors determine the amount of UV radiation that reach certain part of Earth's surface. They are listed and briefly explained below:

• Cloud coverage. Water molecules on clouds scatter the radiation, hence the more clouds the less UV radiation.

,

• Ozone. Similarly as cloud coverage, the more concentration of ozone the less UV radiation that reaches the surface of the Earth.

,

• Angle of incidence. If the angle of incidence of the UV light is oblique the light will spread in a wider area, and hence the intensity is spread across this area.

,

• Aerosols. The molecules of aerosols also scatter the UV light.

,

• Elevation. The more the elevation the greater the amount of UV light.

This question is based on Oscillations and waves. I tried it for days and I just couldn't get it right.

Answers

ANSWER:

The maximun velocity is 16.07 m/s

At x = 0.26

The velocity is 8.36 m/s

The accelearion is 286.67 m/s^2

The resorting force is 86 N

STEP-BY-STEP EXPLANATION:

Given:

k = 310 N / m

Max distance = 0.5 m

Mass of block = 0.3 kg

Max velocity:

Using conservation of energy:

[tex]\begin{gathered} \frac{1}{2}kx^2=\frac{1}{2}mv^2 \\ v^2=\frac{kx^2}{m} \\ \text{ replacing} \\ v^2=\frac{310\cdot0.5^2}{0.3} \\ v=\sqrt[]{258.33} \\ v=16.07\text{ m/s} \end{gathered}[/tex]

At x = 0.26 m:

[tex]\begin{gathered} v^2=\frac{kx^2}{m} \\ v^2=\frac{310\cdot0.26^2}{0.3} \\ v=\sqrt[]{69.85} \\ v=8.36\text{ m/s} \end{gathered}[/tex]

Acceleration:

[tex]\begin{gathered} F=k\cdot x \\ F=m\cdot a \\ \text{ therefore} \\ m\cdot a=k\cdot x \\ a=\frac{k\cdot x}{m} \\ \text{ replacing} \\ a=\frac{310\cdot0.26}{0.3} \\ a=286.67\text{ }\frac{m}{s^2} \end{gathered}[/tex]

The resorting force:

[tex]\begin{gathered} F=m\cdot a \\ \text{ replacing} \\ F=0.3\cdot286.67 \\ F=86\text{ N} \end{gathered}[/tex]

12000 inches to yards

Answers

ANSWER

[tex]\begin{equation*} 333.33\text{ yds} \end{equation*}[/tex]

EXPLANATION

We want to convert 12000 inches to yards.

To do this, divide the value in inches by 36:

[tex]\begin{gathered} 1\text{ in }=\frac{1}{36}\text{ yd} \\ \\ 12000\text{ in }=\frac{12000}{36}\text{ yds }=333.33\text{ yds} \end{gathered}[/tex]

That is the answer.

A can sits on a vertical wooden fencepost 1.9 meters above the ground. Billy picks up a small rock, aims at an angle ϴ = 25⁰ above the horizontal and throws the rock, releasing it 1 m above the ground with an initial speed of v0 =10 m/s. Boom! He hits the can! How far away is the fencepost?

Answers

Given,

Height of the fencepost, h=1.9 m

Angle at which the rock was thrown, θ=25°

The height at which the rock was released, a=1 m

The initial speed of the rock, v₀=10 m/s

Referring to the diagram,

[tex]\tan \theta=\frac{h-a}{d}[/tex]

On rearranging the above equation,

[tex]d=\frac{h-a}{\tan \theta}[/tex]

On substituting the known values,

[tex]d=\frac{1.9-1}{\tan 25^0}=1.93\text{ m}[/tex]

Therefore the fencepost is at a distance of 1.93 m

A massless scaffold is held up by a wire at each end. The scaffold is 12 m long. A300-N box sits 4.0 m from the left end. What is the tension in each wire?1) left wire = 100 N; right wire = 200 N2) left wire = 200 N; right wire = 100 N3) left wire = 900 N; right wire = 2700 N4) left wire = 2700 N; right wire = 900 N

Answers

Free body diagram:

Given data:

Length of massless scaffold (end to end) L=12 m.

Weight of box m=300 N.

Length of massless scaffold (center to end) l=6 m.

As, the box sits 4.0 m from the left end, the distance of the box from the center of massless scaffold is given as,

[tex]\begin{gathered} r=6.0\text{ m}-4.0\text{ m} \\ =2.0\text{ m} \end{gathered}[/tex]

Balancing force in y direction,

[tex]T_1+T_2=300\text{ N}\ldots(1)[/tex]

The torque is given as,

[tex]\tau=perpendicular\text{ distance}\times force[/tex]

Therefore, torque along the center of massless scaffold is given as,

[tex]\begin{gathered} \Sigma\tau=0 \\ l\times T_1+r\times(300\text{ N})-l\times\tau_2=0 \\ 6\times T_1+2\times(300\text{ N})-6\times T_2=0 \\ 6T_1+600\text{ N}-6T_2=0 \\ 6(T_1+100\text{ N}-T_2)=0 \\ T_1+100\text{ N}-T_2=0 \\ T_1-T_2=-100\text{ N}\ldots(2) \end{gathered}[/tex]

Adding equation (1) and (2),

[tex]\begin{gathered} (T_1+T_2)+(T_1-T_2)=300\text{ N}-100\text{ N} \\ T_1+T_1+T_2-T_2=200\text{ N} \\ 2T_1=200\text{ N} \\ T_1=\frac{200\text{ N}}{2} \\ T_1=100\text{ N} \end{gathered}[/tex]

Substituting T1 in equation (1) we get,

[tex]\begin{gathered} 100\text{ N}+T_2=300\text{ N} \\ T_2=300\text{ N}-100\text{ N} \\ T_2=200\text{ N} \end{gathered}[/tex]

Therefore, tension in left wire is 100 N and tension on right wire is 200 N. Hence, option (1) is the correct choice.

Objects with masses M1=12.0 kg and M2= 7.0 kg are connected by a light string that passes over a frictionless pulley as in the figure below. If, when the system starts from rest, M2 falls 1.00m in 1.33s, determine the coefficient of Kinect friction between M1 and the table

Answers

We are asked to determine the coefficient of friction in the sysmtem. First we will do a free body diagram of the first mass, like this:

Where:

[tex]\begin{gathered} N=\text{ Normal Force} \\ m_1=\text{ mass} \\ g=\text{ acceleration of gravity} \\ T=\text{ tension} \\ F_f=\text{friction force} \end{gathered}[/tex]

Now, we add the forces in the horizontal direction, we get:

[tex]T-F_f=m_1a[/tex]

Now, to determine the friction force we need to use the following relationship:

[tex]F_f=\mu N[/tex]

To determine the normal force we will add the forces in the vertical direction. Since there is no movement in the vertical direction this sum must add up to zero:

[tex]\begin{gathered} N-m_1g=0 \\ N=m_1g \end{gathered}[/tex]

Now, we substitute the value of the normal force in the equation for the friction force:

[tex]F_f=\mu m_1g[/tex]

Now, we substitute the friction force in the sum of horizontal forces:

[tex]T-\mu m_1g=m_1a[/tex]

Now, we turn our attention to the second mass. We add the forces in the vertical direction:

[tex]m_2g-T=m_2a[/tex]

Now, since the acceleration "a" and the tension "T" is the same for both masses we will solve fot "T" in the sum of forces for the second mass, like this:

[tex]m_2g-m_2a=T[/tex]

Now, we substitute the value in the sum of forces of the first mass, we get:

[tex]m_2g-m_2a-\mu m_1g=m_1a[/tex]

Now, we solve for the coefficient of friction. To do that we will subtract "m2g" to both sides:

[tex]-m_2a-\mu m_1g=m_1a-m_2g[/tex]

Now, we add "m2a" to both sides:

[tex]-\mu m_1g=m_1a-m_2g+m_2a[/tex]

Now, we divide both sides by "-m1g":

[tex]\mu=\frac{m_1a-m_2g+m_2a}{-m_1g}[/tex]

We have gotten an expression for the coefficient of friction but we need to determine the acceleration of the system. To do that we will use the fact that the second mass moves 1 meter in 1.33 seconds. Assuming constant acceleration we can use the following equation of motion:

[tex]y=v_0t+\frac{1}{2}at^2[/tex]

Where:

[tex]\begin{gathered} y=\text{ distance} \\ v_0=\text{ initial velocity} \\ t=\text{ time} \\ a=\text{ acceleration} \end{gathered}[/tex]

Since the mass starts from rest we have that the initial velocity is zero, therefore:

[tex]y=\frac{1}{2}at^2[/tex]

Now, we solve for the acceleration. First, we multiply both sides by 2:

[tex]2y=at^2[/tex]

Now, we divide both sides by the time squared:

[tex]\frac{2y}{t^2}=a[/tex]

Now, we plug in the values:

[tex]\frac{2(1m)}{(1.33s)^2}=a[/tex]

Solving the operations:

[tex]1.13\frac{m}{s^2}=a[/tex]

Now, we substitute the values in the equation for the coefficient of friction:

[tex]\mu=\frac{(12kg)(1.13\frac{m}{s^2})-(7kg)(9.8\frac{m}{s^2})+(7kg)(1.13\frac{m}{s^2})}{-(12kg)(9.8\frac{m}{s^2})}[/tex]

Solving the operations we get:

[tex]0.4=\mu[/tex]

Therefore, the coefficient of friction is 0.4

You wish to lift a 720N crate of bricks to the 3rd floor of a building in a construction site. The 3rd floor is 16m high. How much work will that require?

Answers

ANSWER

[tex]11,520J[/tex]

EXPLANATION

Parameters given:

Weight (force), F = 720N

Height (distance), d = 16m

To find the work required to lift the crate to the 3rd floor, we have to find the product of the force (weight of the crate) and the distance it will be lifted.

Therefore, we have:

[tex]\begin{gathered} W=F\cdot d \\ W=720\cdot16 \\ W=11,520J \end{gathered}[/tex]

That is the work that it will require.

a given mass of a gas at - 73°C exerts pressure of 40 cm of mercury. what pressure will be exerted at 127 °C if the volume remains constant

Answers

The pressure that will be exerted at 127 °C if the volume remains constant is 80 cm of mercury

How do I determine the new pressure?

First, we shall list out the given parameters from the question. This is shown below:

Initial temperature (T₁) = -73 °C = -73 + 273 = 200 K Initial pressure (P₁) = 40 cm of mercuryNew temperature (T₂) = 127 °C = 127 + 273 = 400 KVolume = ConstantNew pressure (P₂) = ?

Thus, we can obtain the new pressure of the gas at 127 °C as illustrated below:

P₁ / T₁ = P₂ / T₂

40 / 200 = P₂ / 400

Cross multiply

200 × P₂ = 40 × 400

200 × P₂ = 16000

Divide both sides by 200

P₂ = 16000 / 200

P₂ = 80 cm of mercury

Therefore, the new pressure is 80 cm of mercury

Learn more about gas laws:

https://brainly.com/question/15343985

#SPJ1

A radioactive tracer has a half-life of two hours how much of a 2500 g sample will be available after 18 hours?

Answers

ANSWER:

4.88 grams

STEP-BY-STEP EXPLANATION:

We must first calculate how many half-life there are in 18 hours, knowing that each half-life takes 2 hours.

[tex]\frac{18}{2}=9\text{ half-life}[/tex]

Now, knowing this, we can calculate the number of grams applying 9 times the half-life, like this:

[tex]\frac{2500}{2^9}=4.88\text{ g}[/tex]

Which means that after 18 hours there are 4.88 grams

A 0.2-kg aluminum plate, initially at 20°C, slides down a 15-m-long surface, inclined at a 30°angle to the horizontal. The force of kinetic friction exactly balances the component ofgravity down the plane so that the plate, once started, glides down at constant velocity. If90% of the mechanical energy of the system is absorbed by the aluminum, what is itstemperature increase at the bottom of the incline? (Specific heat for aluminum is 900J/kg⋅°C.) Why do I multiply 15 by sin30?

Answers

A scheme of a the given situation is shown below:

First, consider that the work over the plate is done only by the component of the weight parallel to the incline (due to the perpendicular component is balanced by the friction force), then, the work on the plate is:

W = m*g*d*sinθ

where,

m: mass = 0.2kg

d: length of the incline = 15m

g: gravitational acceleration constant = 9.8m/s^2

θ = 30

By replacing the previous values into the expression for W, you obtain:

W = (0.2 kg)(9.8 m/s^2)(15 m)sin(30)

W = 14.7 J

Now, take into account that the amount of heat absorbed by the aluminum plate is given by the following formula:

Q = m*c*ΔT

Q: heat

m: mass

c: specific heat

ΔT: change in tempetaure

Take into account that the 90% of the mechanical energy is absorbed by the plate, which means that 0.9 of the work is converted to absorbed heat by the plate.

Then, you can write:

0.9W = Q

0.9(14.7J) = Q

13.23J = Q

Replace the given expression for Q into the previous equation and solve for ΔT, as follow:

m*c*ΔT = 13.23 J

ΔT = 13.23J/(m*c)

Now, replace the values of m and c for aluminum and simplify:

ΔT = 13.23J/(0.2kg*900J/kg°C)

ΔT = 0.0735°C

Hence, the temperature increase at the bottom of the incline is approximately 0.07°C

A truck can travel at 100 km/hr How long would it take to drive 900km?

Answers

Given:

The speed of the truck is,

[tex]v=100\text{ km/hr}[/tex]

The distance is,

[tex]s=900\text{ km}[/tex]

The time to drive this distance is,

[tex]t=\frac{s}{d}[/tex]

Substituting the values we get,

[tex]\begin{gathered} t=\frac{900}{100} \\ =9\text{ hrs} \end{gathered}[/tex]

Hence, the time is 9 hrs.

If you have a convex lens whose focal length is 10.0 cm, where would you place an object in order to produce an image that is virtual?

Answers

When an object is placed between first focus and optical center of a convex lens then virtual image is produced.

Here , focal length is 10.0 cm . So object distance should be less than 10.0 cm

Final answer is : between focus and optical center of the lens

A 10 gram ball is rolling at 3 m/s. Calculate its kinetic energy.

Answers

ANSWER:

0.045 joules

STEP-BY-STEP EXPLANATION:

Given:

mass (m) = 10 g = 0.01 kg

velocity (v) = 3 m/s

The kinetic energy is given by the following formula:

[tex]K_E=\frac{1}{2}mv^2[/tex]

We replacing:

[tex]\begin{gathered} K_E=\frac{1}{2}\cdot0.01\cdot3^2 \\ K_E=0.045\text{ J} \end{gathered}[/tex]

The kinetic energy is 0.045 joules.

A human heart found to beat seventy five times in a minute. Calculate the beat frequency?​

Answers

[tex]{ \green{ \tt{f = \frac{number \: of \: beats}{time \: taken}}}} [/tex]

[tex]{ \green{ \tt{number \: of \: beats = 75}}}[/tex]

[tex]{ \green{ \tt{time \: taken =1 \: min \: = 60 \: sec}}}[/tex]

[tex]{ \red{ \sf{f = \frac{ \cancel{75^{3}}}{ \cancel{ 60_{4} }}}}}[/tex]

[tex]{ \blue{ \boxed{ \purple{ \sf{f = \frac{3}{4} = 1.2 {s}^{ - 1}}}}}} [/tex]

___________________________________

[tex]{ \blue{ \sf{T = \frac{1}{f}}}} [/tex]

[tex]{ \blue{ \sf{T = \frac{1}{ \purple{ \sf{1.2}}}}}} [/tex]

[tex]{ \boxed{ \red{ \sf{T = 0.8 \: S}}}}[/tex]

i need help asap i just need the vocab i need help with this please help

Answers

ANSWERS

0. First law of Thermodynamics:, ,B,. Thermal energy can change form and location, but it cannot be created or destroyed

,

1. Thermal Energy: ,H,. Kinetic energy in transit from one object to another due to temperature difference

,

2. Temperature: ,D,. the average kinetic energy of particles in an object - not the total amount of kinetic energy particles

,

3. Absolute zero: ,J, occurs when all kinetic energy is removed from an object

,

4. Thermal Equilibrium: ,E, obtained when touching objects within a system reach the same temperature

,

5. Zeroth Law of Thermodynamics: ,C, if two systems are separately found to be in thermal equilibrium with a third system, the first two systems are in thermal equilibrium with each other.

,

6. Conduction: ,F, ,the transfer of thermal energy within an object or between objects from molecule to molecule

,

7. Coefficient of Heat Conductivity: ,G, is the measure of a material's ability to conduct heat.

,

8. Thermal resistance of a material: ,A, the measure of a material's ability to resist heat

,

9. Radiation: ,I, the process by which energy is transmitted through a medium, including empty space, as electromagnetic waves.

EXPLANATION

The zeroth and first laws of thermodynamics are known statements, as well as the definition of thermal equilibrium, so there is nothing to explain about these three.

Thermal energy is a form of kinetic energy. It is "generated" by the vibrations of the molecules or atoms that form the object, causing the object to increase its temperature. As a result, the temperature is the average kinetic energy of these particles.

Absolute zero is a temperature of 0 degrees Kelvin - approximately -273 degrees Celsius. When an object reaches this particular temperature it is considered as if the object has "no temperature" - that is why it is called absolute zero. As explained above, the temperature is the average kinetic energy of the object's particles, so if the particles have no kinetic energy, then the object has zero temperature.

Heat conductivity and thermal resistance are basically opposites. The first is the object's ability to conduct heat while the second is the ability to resist the conduction of heat.

Radiation and Conduction are two ways energy can be transferred. The first does not need a medium - that is why energy can be transmitted through empty space, and the second is by contact between two objects or an object and a medium.

I need help with #2 it off s for practice

Answers

First, find the acceleration using Newton's Second Law.

[tex]\begin{gathered} F=ma \\ a=\frac{F}{m} \end{gathered}[/tex]

Where F = 8.10x10^5 N and m = 1.40x10^7 kg.

[tex]a=\frac{8.10\times10^5N}{1.40\times10^7\operatorname{kg}}=5.79\times10^{-2}\cdot\frac{m}{s^2}[/tex]

Then, use a formula that relates acceleration, initial velocity, final velocity, and time.

[tex]v_f=v_0+at[/tex]

Solve for t because the problem is asking to find the time.

[tex]\begin{gathered} v_f-v_0=at_{} \\ t=\frac{v_f-v_0}{a} \end{gathered}[/tex]

Where vf = 64 km/h, v0 = 0, and a = 5.79x10^-2 m/s^2. Before we continue, we need to transform the final velocity to m/s.

[tex]v_f=64\cdot\frac{km}{h}\cdot\frac{1000m}{1\operatorname{km}}\cdot\frac{1h}{3600\sec}=17.78\cdot\frac{m}{s}[/tex]

Once we have the velocity transformed, we are able to find t.

[tex]\begin{gathered} t=\frac{17.78\cdot\frac{m}{s}-0}{5.79\times10^{-2}\cdot\frac{m}{s^2}} \\ t=3.07\times10^2\sec \\ t=307\sec \end{gathered}[/tex]

But, the answer must be in minutes.

[tex]t=307\sec \cdot\frac{1\min}{60\sec}=5.12\min [/tex]

Therefore, it takes 5.12 minutes.

I was told it was 5.886 J by another tutor on here but that was incorrect so just trying again

Answers

The potential energy of gravity is given by:

[tex]\begin{gathered} E=mgh \\ where: \\ m=0.3 \\ h=2 \\ g=9.8 \\ so: \\ E=0.3\cdot2\cdot9.8 \\ E\approx5.9J \end{gathered}[/tex]

Answer:

5.9 J

Question 28 of 30The graph below shows the conservation of energy for a skydiver jumping outof a plane and landing safely on the ground. Which energy is represented byline A?сAEnergy (kJ)BDistance (m)A. Potential energyB. Kinetic energy

Answers

Answer:

Explanation:

Potential energy depends on the height of the object above the ground. As the height decreases, the potential energy decreases. Looking at line A, the energy is decreasing as the distance is increasing. Thus, the energy represented by line A is

A. Potential energy

A concave mirror of focal length 10 cm forms an upright and diminished image of a real object placed at a distance of 5 cm from the mirror. Is this true or false?

Answers

Given,

The focal length of the concave mirror, f=10 cm

The object distance, o=5 cm

From the mirror formula, we have,

[tex]\frac{1}{f}=\frac{1}{o}+\frac{1}{i}[/tex]

Where i is the image distance.

On substituting the known formula,

[tex]\begin{gathered} \frac{1}{10}=\frac{1}{5}+\frac{1}{i} \\ \Rightarrow\frac{1}{i}=\frac{1}{10}-\frac{1}{5} \\ =-\frac{1}{10} \\ \Rightarrow i=-10\text{ cm} \end{gathered}[/tex]

And the magnification is given by,

[tex]m=\frac{-i}{o}[/tex]

On substituting the known values,

[tex]\begin{gathered} m=\frac{-(-10)}{5} \\ =2 \end{gathered}[/tex]

Thus the object is not diminished, it is magnified.

Thus the given statement is false.

A solenoid is wound with 259 turns per cm. An outer layer of insulated wire with 51 turns per cm is wound over the first layer of wire. The inner coil carries a current of 7.577 A, and the outer coil carries a current of 21.68 A in the opposite direction. What is the magnitude of the magnetic field, in microTeslas, at the central axis ?

Answers

Answer tab

For this question, we'll first define a number, called the "linear density of the coil" which is exactly the one the exercise gives us, in a unit of turns per length. The magnetic field can be calculated as:

[tex]B=\mu_0in[/tex]

Where n is the linear density.

In our case, as the coils carry current in opposite directions, the generated magnetic fields will be opposed, and we'll have:

[tex]B=B_{outer}-B_{inner}=\mu_0*(51*10^2*21.68-259*10^2*7.577)[/tex]

Please note that we had to multiply by 10^2, in order to convert turns/cm to turns/m

Then, our final magnetic field will be:

[tex]|B|=0.107664T=107664\mu T[/tex]

Our final answer is B=107664uT

If a rock has 376 J of potential energy when it’s held 10.1 m above the ground what is its mass? Round to the nearest tenth

Answers

Answer: 3.8 kg

Explanation:

The formula for calculating potential energy is expressed as

Potential energy = mgh

where

m is the mass of the object

g is the acceleration due to gravity

h is the height above the ground

From the information given,

Potential energy = 376

g = 9.8 m/s^2

h = 10.1

Thus,

376 = m x 9.81 x 10.1

376 = 99.081m

m = 376/99.081

m = 3.8 kg

The mass is 3.8 kg

A diagram of a student on a playground swing is shown below. At which point is the kinetic energy the greatest?A. Point 1B. Point 2C. Point 3D. Point 4

Answers

To find:

The point where the kinetic energy is the greatest.

Explanation:

The kinetic energy of an object is the energy possessed by the object due to its motion. Thus the kinetic energy of an object is proportional to the square of the velocity of the object.

The velocity of the wing will be zero at the extreme points of the path of the swing. But as the swing approaches the mean point, i.e., point 3, the velocity of the swing increases. The velocity will be maximum at the mean point.

Thus the kinetic energy will be the greatest at point 3.

Final answer:

Therefore the correct answer is option C.

You yell down a very deep well and it takes 1.5 s for your echo to return. If the speed of sound is 340 m/s then how deep is the well ?

Answers

We know that

• The time of the echo to return is 1.5 seconds.

,

• The speed of the sound is 340 m/s.

It's important to consider that the sound wave has a constant speed, that is, it doesn't change its velocity. Therefore, we have to use the relation

[tex]d=v\cdot t[/tex]

Where t = 1.5 sec and v = 340 m/s. Let's find d

[tex]\begin{gathered} d=340m/s\cdot1.5\sec \\ d=510m \end{gathered}[/tex]Hence, the well is 510 meters deep.

I need help with this table pleasecalculate relative density of steel. Use table 3

Answers

Take into account that the relative density is given by:

[tex]\rho_{\text{rel}}=\frac{\rho}{\rho_{\text{water}}}[/tex]

where ρ, in this case, is the density of the steel and ρwater is the density of water (1000 kg/m^3).

The density of the steel is:

[tex]\rho=\frac{\text{mass}}{\text{volume}}[/tex]

Based on table 3, you have:

mass = 50.7 g = 0.0507 kg

volume = 0.0000063 m^3

[tex]\rho=\frac{0.0507kg}{0.0000063m^3}\approx8047.62\frac{kg}{m^3}[/tex]

Then, for the relative density you obtain:

[tex]\rho_{\text{rel}}=\frac{\rho}{\rho_{\text{water}}}=\frac{8047.62\frac{kg}{m^3}}{1000\frac{kg}{m^3}}\approx8.048[/tex]

Hence, the relative density of steel is 8.048

A wheel Was spinning at 2.8 rad/s. It took 3.2 seconds to stop completely. What is the acceleration of the wheel?

Answers

Given

The angular velocity is

[tex]\omega=2.8\text{ rad/s}[/tex]

The time taken,

[tex]t=3.2s[/tex]

To find

The acceleration of the wheel

Explanation

The acceleration is

[tex]\begin{gathered} \alpha=\frac{\omega}{t} \\ \Rightarrow\alpha=\frac{2.8}{3.2}=\frac{0.875rad}{s^2} \end{gathered}[/tex]

Conclusion

The acceleration is

[tex]0.875\text{ rad/s}^2[/tex]

Other Questions
Elena is conduction a study about the effects of toxins in the water on the hormones of fish. Elena surveys 350 male fish in a river and finds that 150 of the male fish have egg cells growing inside them. According to Elenas survey, what is the ratio of male fish with egg cells to male cells in the river? For the following exercise, find the horizontal intercepts, the vertical intercept, the vertical asymptotes, and the horizontal or slant asymptotes of the function. use that information to sketch graph. Can you please help me out with a question Part D Which details from the text best help to develop the key idea in Part C? Select the two correct answers. Question 4 options: "My mother had never gone inside of a schoolhouse, and so she was not capable of comforting her daughter who could read and write." "It was next to impossible to leave the iron routine after the civilizing machine had once begun its day's buzzing;" "I took it from her hand, for her sake; but my enraged spirit felt more like burning the book, which afforded me no help, and was a perfect delusion to my mother." "[My mother's few words hinted that I had better give up my slow attempt to learn the white man's ways, and be content to roam over the prairies and find my living upon wild roots." The equation of the line of best fit is y= 25x+7.5. What does the y-intercept represent? dr. liu participates in a police ride-along program in order to examine the day-to-day activities of police officers. this method of gathering information may best be described as a(n) . Analyze the data in the line plot "attached "Use the data to construct a line plot.Number of students in a classroom:22, 28, 31, 33, 28, 29, 31, 28, 29, 32, 27, 18, 29, 31, 30, 31, 32, 27, 29, 33 which type of love, according to sternberg, consists of passion and intimacy (with no commitment) and generally refers to the statement, "i'm in love"? D=2730 mi t=9.75 h find r Harry took a loan from the bank.D represents Harry's remaining debit (In dollars) after t months.D = -200t + 9000 The function shown by the graph is non-differentiable at x = 4. Why?A. It has a vertical tangent line at x = 4.B. It has a jump discontinuity at x = 4.C. It has a corner at x = 4.D. It has a vertical asymptote at x = 4.Answer: In photo belowYou're Welcome :) Hello, I needed help with a few assignment questions. Thank you. It's about Newton's second law of motion. Can you please help me with the answers, I dont need the work I just need the answer please Rotation of 270 (x,y) or (a,b) becomes What is the domain of h?y7h---7 6 5 4 3 276+5+4+3+21++ +++++1 2 3 4 5 6 72+-3+-4+-5 .-6+-7 A rocket accelerates at 65 m/s with the force of 54,080 N. What is the mass of the rocket? Near the top of the Citigroup Center building in New York City, there is an object with mass of 4.00 105 kg on springs that have adjustable force constants. Its function is to dampen wind-driven oscillations of the building by oscillating at the same frequency as the building is being driventhe driving force is transferred to the object, which oscillates instead of the entire building.(a) What effective force constant (in N/m) should the springs have to make the object oscillate with a period of 2.10 s? N/m(b) What energy (in J) is stored in the springs for a 1.80 m displacement from equilibrium? J if the great circle circumference of a sphere is 16 pi yards, find its surface area. How was Whitman's poor upbringing valuable for Americans at a time when most of the writing they read was written by wealthy Americans? Susan monitors the number of strep infections reported in a certain neighborhood in a given week.The recent numbers are shown in this table:WeekNumber of People020126234344According to her reports, the reported infections are growing at a rate of 30%.If the number of infections continues to grow exponentially, what will the number of infections bein week 10?